Točno
26. siječnja 2016. 20:39 (8 godine, 3 mjeseci)
If a, b, c are three positive real numbers such that ab+bc+ca = 1, prove that \sqrt[3]{ \frac{1}{a} + 6b} + \sqrt[3]{\frac{1}{b} + 6c} + \sqrt[3]{\frac{1}{c} + 6a } \leq \frac{1}{abc}.
Upozorenje: Ovaj zadatak još niste riješili!
Kliknite ovdje kako biste prikazali rješenje.

Ocjene: (2)



Komentari:

Aha, da, nije komplicirano. Hvala!

Najednostavije je ab = x, bc = y, ca = z, pa nejednakost jednostavno postane : (x + y + z)^2 >= 3(xy + yz + zx)

Kako se najjednostavnije pokaže zadnja tvrdnja (ab + bc + ca)^2 \geq abc(a + b + c)?
Pretpostavljam da postoji nešto očito, ali ne pada mi na pamet ništa osim raspisivanja i A-G po dijelovima.
Najednostavije je ab = x, bc = y, ca = z, pa nejednakost jednostavno postane : (x + y + z)^2 >= 3(xy + yz + zx)

Kako se najjednostavnije pokaže zadnja tvrdnja (ab + bc + ca)^2 \geq abc(a + b + c)?
Pretpostavljam da postoji nešto očito, ali ne pada mi na pamet ništa osim raspisivanja i A-G po dijelovima.
Kako se najjednostavnije pokaže zadnja tvrdnja (ab + bc + ca)^2 \geq abc(a + b + c)?
Pretpostavljam da postoji nešto očito, ali ne pada mi na pamet ništa osim raspisivanja i A-G po dijelovima.
bas lijepo rjesenje!
inace, morao sam guglati da se uvjerim u iskaz holdera: (abc+def+ghi)^3 \leq (a^3+d^3+g^3)(b^3+e^3+h^3)(c^3+f^3+i^3) pa stavljam ovdje ako jos neko zapne citajuci taj korak.